1987 AJHSME Problems/Problem 3

Revision as of 20:38, 23 November 2016 by AlphaPi17 (talk | contribs) (Solution 2)

Problem

$2(81+83+85+87+89+91+93+95+97+99)=$

$\text{(A)}\ 1600 \qquad \text{(B)}\ 1650 \qquad \text{(C)}\ 1700 \qquad \text{(D)}\ 1750 \qquad \text{(E)}\ 1800$

Solution 2

Find that \[(81+83+85+87+89+91+93+95+97+99) = 5 \cdot 180\] Thus \begin{align*} 2(5 \cdot 180) &= 10 \cdot 180\\ &= 1800 & \text{ Thus (E) is the correct answer} \end{align*}

Solution 1

Find that \[(81+83+85+87+89+91+93+95+97+99) = 5 \cdot 180\] Thus \begin{align*} 2(5 \cdot 180) &= 10 \cdot 180\\ &= 1800 & \text{ Thus (E) is the correct answer} \end{align*}

See Also

1987 AJHSME (ProblemsAnswer KeyResources)
Preceded by
Problem 2
Followed by
Problem 4
1 2 3 4 5 6 7 8 9 10 11 12 13 14 15 16 17 18 19 20 21 22 23 24 25
All AJHSME/AMC 8 Problems and Solutions

The problems on this page are copyrighted by the Mathematical Association of America's American Mathematics Competitions. AMC logo.png